MCQingeneralsurgeryforundergraduates PDF
MCQingeneralsurgeryforundergraduates PDF
MCQingeneralsurgeryforundergraduates PDF
net/publication/315956583
CITATIONS READS
0 3,211
2 authors:
Some of the authors of this publication are also working on these related projects:
Epidemiology and predictors of recurrence of Marjolin’s ulcer: experience from Mansoura University View project
All content following this page was uploaded by Islam Hany on 27 September 2018.
These questions have been collected and selected from the most updated and
recent international books in general and special surgery.
The main format and presentations of the questions have been exclusively
revised to coincide with the recommended topics.
If you have any comments as to the contents or the usefulness of this book, send
them to my e-mail address: [email protected]
Thanks,
Mansoura University
1
Vascular
1. About Claudication which of the following statements are true?
a. Doppler ultrasound works on the basis of a frequency shift when sound waves hit
moving red blood cells.
b. ABPI means ‘ankle brachial pulsatility index.
c. An ABPI of greater than 0.9 is probably normal.
d. Duplex scans are a combination of wave form analysis and B-mode ultrasound.
e. Carotid surgery is often performed on the basis of duplex scans without further imaging.
3. Drugs which of the following drugs have been shown to improve claudication?
a. Beta-blockers
b. Aspirin
c. Oxpentifylline
d. Simvastatin
e. Prostacyclin.
a. Buerger’s disease
b. Infection
c. Intra-arterial drug injection
d. Frostbite
e. Deep vein thrombosis insufficiency.
1 2 3 4
C B C E
2
5. About amputation, which of the following are not an indication for major
amputation?
5 6 7 8
C C B C
3
9 10 11 12
B D C D
4
15. About acute arterial thrombosis all the following ore true except:
a. Old age
b. History of chronic ischemia
c. No collateral
d. There may be history of diarrhea
16. Sure signs of arterial injury include all the following except:
a. Signs of ischemia
b. Evidence of adjacent nerve injury
c. Pulsating hematoma
d. Palpable thrill at the site of injury
13 14 15 16
C C C B
5
18. Management of complete arterial rear may include all the following except
a. Dissection of the artery
b. Cut the minor bronchus
c. Suturing in transverse suture line
d. Saphenous grafting
17 18 19 20
D C d B
6
23. A 55-year-old woman gives a history of tiredness, aching, and a feeling of heaviness in the
left lower leg for the past 3 months. These symptoms are relieved by leg elevation. She is also
awakened frequently by calf and foot cramping, which is relieved by leg elevation, walking, or
massage. On physical examination there are superficial varicosities, non pitting edema, and a
slightly painful, 2 cm. diameter superficial ulcer 5 cm. above and behind the left medial malleolus.
What is the most appropriate diagnosis?
24. The best treatment plan for the patient described in the preceding question should
include:
21 22 23 24
B D C e
7
25. In patients who develop a documented episode of deep venous thrombosis (DVT) the most
significant long-term sequale is:
a. Claudication.
b. Recurrent foot infection s.
c. Development of stasis ulcer.
d. Pulmonary embolization.
e. Diminished arterial perfusion.
26. A 28-year-old woman developed a painful thrombosis of a superficial varix in the left
upper calf 2 days previously. After spending the 2 days in bed with her leg elevated, she felt
better and the tenderness resolved; however, when out of bed she developed a twinge of
right-sided chest pain when walking and a feeling of heaviness in the calf. Which treatment is
most appropriate?
a. Check for leg swelling, tenderness, and Homan's sign, and obtain a Doppler ultrasound study.
b. Begin antibiotics for a probable secondary bacterial infection.
c. Order emergency venography, and if it is abnormal, begin heparin administration.
d. Begin ambulation and discontinue bed rest that probably caused muscle pain by hyperextension of the
knee.
e. If there is no pain on dorsiflexion of the left foot reassure her, since a negative Homan's sign precludes the
diagnosis of DVT.
27. In a 55-year-old grocery store cashier with an 8-month history of leg edema increasing over the
course of a work day, associated with moderate to severe lower leg bursting pain, the most
appropriate investigative study or studies are:
28. In an otherwise healthy male with previously normal pulmonary and cardiac function,
how much of the pulmonary vascular bed must usually be occluded to produce an unstable
cardiovascular state (shock)?
a. 10%.
b. 20%.
c. 40%.
d. More than 50%.
25 26 27 28
C C A d
8
30. The single most important indication for emergency pulmonary embolectomy is:
31. In prevention of the fat emboli syndrome the primary therapy can be
accomplished by which of the following?
29 30 31 32
C c d c
9
34. Which of the following con firms the diagnosis of transection of the descending
thoracic aorta?
a. Widened mediastinum.
b. Fractured first rib.
c. Left pleural effusion.
d. Positive aortogram.
e. All of the above.
36. The most common risk associated with carotid artery aneurysm is:
33 34 35 36
c d C b
10
a. Sepsis.
b. A congenital defect.
c. Atherosclerosis.
d. Fibromuscular dysplasia.
a. Celiac.
b. Superior mesenteric.
c. Hepatic.
d. Splenic.
a. Matas.
b. Linton.
c. Dubost.
d. Non e of the above.
37 38 39 40
c d b c
11
42. The appropriate treatment in most situations of an aortic abdominal graft that
has become infected is:
43. In a patient with an abdominal aortic aneurysm and a history of several previous
abdominal procedures for release of dense peritoneal adhesions causing episodes
of intestinal obstruction, consideration should be given to which one of the
following at operation?
a. Cardiopulmonary bypass.
b. An incision from the xiphoid process to the symphysis pubis.
c. Incision in the left flan k with a retroperitoneal approach.
d. An axillobifemoral graft.
44. The incidence of inflammatory aortic abdominal aneurysms with dense periaortic
adhesions and possible involvement of adjacent structures such as the duodenum,
renal vein, and ureter is approximately:
a. 2%.
b. 10%.
c. 25%.
41 42 43 44
a d c b
12
45. Carotid artery occlusive disease most often produces transient ischemic
attacks or stroke by which of the following mechanisms?
a. Reduction of flow to the affected area of the brain through stenotic or occluded
vessels.
b. Embolization of atheromatous debris and/or clot with occlusion of intracranial
branches of the carotid artery.
c. Thrombosis and propagation of the clot into the intracranial branches.
d. All of the above are equally common.
47. In terms of long-term graft patency, the best results in the femoral tibial bypass
position have been achieved with:
48. Which of the following statements about femoral popliteal bypass grafting is/are
true?
a. Lon g-term graft surveillance by duplex scanning has no effect on graft patency
rates.
b. Graft failure and amputation occur in half the patients within 5 years.
c. If grafting is successful, long-term mortality is improved.
d. Patency rates of 80% to 90% at 1 year currently are expected.
45 46 47 48
B C D d
13
49. Which of the following statements about percutaneous renal artery transluminal angioplasty
(PRTA) are true?
a. Patients with renovascular hypertension are usually cured after successful PRTA.
b. Patients with renovascular hypertension due to atherosclerosis are more likely to benefit
from PRTA than those in whom it is due to fibromuscular dysplasia.
c. PRTA of ostial atherosclerotic lesions is more successful than PRTA of nonostial lesions.
d. PRTA is associated with a higher morbidity and mortality than angioplasty for peripheral
vascular disease.
51. A patient presents with a gunshot wound of the mid-neck. Although drunk, he exhibits no
lateralizing neurologic signs. After control of his airway is achieved, he is taken directly to the
operating room for control of hemorrhage. The common carotid artery has a 2-cm. destroyed
segment. There is also a major esophageal injury. The best treatment for this carotid injury is:
52. A 35-year-old man involved in a motor vehicle accident presents with a knee
dislocation that is easily reduced. Radiography of the knee shows no fracture. Which of the
following statements about h is treatment are true?
49 50 51 52
D d a b
14
a. Explore all the major arteries and veins of the pelvis and surgically control the
bleeding if possible.
b. Do not explore the pelvic hematoma. Close the abdomen and apply a MAST suit.
c. Do not explore the pelvic hematoma. Apply a pelvic fixator and send the patient
to radiology f or possible embolization of bleeding pelvic vessels.
d. Use sustained hypotensive anesthesia to try to control bleeding.
e. Open the pelvic hematoma and apply laparotomy pads with topical hemostatic
agents.
54. Which of the following statements about acute arterial occlusion today is/are not
true?
55. Which of the following statements about lytic agents is/are true?
a. They were first introduced well after the advent of balloon embolectomy.
b. Streptokinase is nonantigenic.
c. Systemic use is the most effective means of delivery.
d. The interval to reperfusion limits their utility in the treatment of advanced
ischemia.
53 54 55 56
C d d d
15
57. Which of the following is/are not true of the embolectomy catheter technique?
a. The balloon should be inflated by the same person who withdraws the catheter.
b. Distal exploration should be carried out in all major branches of the affected
extremity.
c. The balloon is designed to dilate as it traverses areas of luminal narrowing.
d. Removal of adherent thrombus requires alternate catheter-based therapy in
addition to balloon exploration.
58. Which of the following is the least reliable indicator of successful thrombectomy?
59. A 21-year-old woman presents with digital color changes in response to cold
stimulation. Physical examination and laboratory data, including an autoimmune
disease screen, are normal. She should be advised that:
57 58 59 60
C a a d
16
61. The most dangerous DVT which requires the longest period of treatment:
62. Which of the following statements about upper extremity edema is/are true?
63. Which of the following statements about chronic mesenteric ischemia due to
atherosclerosis is/are correct?
a. It is a decapeptide.
b. It is an enzyme found in high concentration in the pulmonary circulation.
c. It is a direct vasoconstrictor and stimulates aldosterone production.
d. It is a vasoconstrictor and inhibits aldosterone secretion.
61 62 63 64
c b e c
17
65. Which of the following statements about atherosclerotic obstruction of the renal arteries
is true?
66. Which of the following statements about the treatment of renal artery stenosis is/are true?
a. Though a significant cause of hypertension, renal artery stenosis seldom results in loss
of renal function.
b. In patients with medically controlled renovascular hypertension there is no need to
consider revascularization.
c. Balloon angioplasty is more effective in patients with atherosclerotic disease as
compared with those with fibromuscular disease.
d. In patients with severe atherosclerosis of the aorta, bypass from the splenic or hepatic
arteries should be considered.
67. All but one of the following statements is true. Which is not true?
a. Successful clinical repair of injured veins had been affected by the turn of the twentieth
century.
b. Initial large experience in managing injured vein s came from the battlefields of
twentieth-century wars.
c. More than 50% of repaired injured veins thrombose.
d. Phlebography is useful in evaluating variable venous anatomy.
e. Repeated phlebography following attempted venous repair is useful in determining the
success rate.
68. Concerning in-vivo regulation of the anticoagulated state by endothelium, the following
is/are true:
65 66 67 68
a d c d
18
69. Ten years after irradiation of the neck for a tonsillar carcinoma, a 59-year-old
woman is found to have symptomatic carotid artery disease. Arteriogram shows a 70%
irregular stenotic lesion. The following is/are true:
a. Replacement of the artery should be planned due to radiation induced arterial injury
b. The pathology is most likely to be an inflammatory reaction with endothelial
sloughing and thrombosis
c. If atherosclerotic disease is found, the plaque will be no different than nonirradiated
plaques
d. The patient should be managed medically because of the radiation arterial in jury
70. A 23-year-old woman with fever, myalgia and anorexia presents with hypertension
and a cool, ischemic left arm. Angiography shows multiple stenoses of the subclavian and
renal arteries. The following is/are true:
a. Plasminogen is an a-globulin
b. Fibrin but not fibrinogen is lysed by plasmin
c. The main inhibitor of plasmin is a2-macroglobulin
d. TAP is activated during fibrin bonding to plasminogen
72. An 82-year-old man with a long history of coronary and peripheral vascular disease
presents with an acutely ischemic right lower extremity. The following is/are true:
69 70 71 72
C D D b
19
73. Two days following coronary angiography and angioplasty, a 47-year-old male diabetic
develops painful blue toes on both feet. The following is/are true:
a. It is very unlikely that there is any connection between the catheterization and the
extremity problem
b. The appropriate treatment is vasodilators and an antiplatelet agent
c. If both superficial femoral arteries are obstructed, the most likely etiology is in-situ
microvascular thrombosis
d. If renal failure or pancreatitis develops, the outlook for long term survival is very poor
74. A 70-year-old man presents with sudden pain and ischemic changes in his left leg.
An arterial embolus is suspected. The following is/are true:
75. A 67-year-old man with acute popliteal arterial embolism has a negative cardiac echo for
source of the thrombus. The following is/are true:
76. Which of the following is/are appropriate candidates for exercise testing?
a. The patient with symptoms of intermittent claudication but normal resting ankle
brachial indices
b. The patient with rest pain, non healing ulcers or gangrene
c. If the resting ankle pressure is below 30–40 mmHg
d. The patient with blue toe syndrome and readily palpable pedal pulses
73 74 75 76
D C c a
20
77. Which of the following statements is/are true regarding the use of duplex
scanning as a means to follow and monitor bypass grafts?
a. Strict control with insulin but not oral hypoglycemic agents markedly reduces
the incidence of cardiovascular complications in diabetic persons
b. Vascular complications are directly proportion al to the degree of glycemic
control
c. The effects of diabetes are most marked in individuals with other risk factors
d. The impact of diabetes and cardiovascular risk is relatively uniform
77 78 79 80
A c c c
21
83. About phlegmasia cerulae dolens all the following ore true except:
a. There is massive iliofemoral DVT
b. Severe congestion and cyanosis of lower limb
c. Treated by anticoagulants
d. May lead to venous gangrene
81 82 83
D D C
22
1) 2) 3) 4)
B A B D
23
5- Which one among the following statements about cystic hygroma is untrue?
a. Affects infants and young children.
b. Occurs chiefly in the neck, axilla and groin.
c. Presents as a large soft fluctuating translucent swelling.
d. is localized to the subcutaneous tissues.
e. May rupture subcutaneously with spontaneous cure.
6-The false statement about ranula is that it:
a. is a bluish cyst in the angle between the tongue and the floor of mouth.
b. Usually lies to one side of the middle line.
c. May extend into the submandibular region.
d. May assume an hour-gloss appearance.
e. is best treated by complete excision.
7. Which statement about salivary calculi is untrue?
a. Are commonest in the parotid gland.
b. May lie in the gland or its duct.
c. Produce recurrent pain and swelling during meals.
d. Contain a high proportion of calcium.
e. May require excision of the affected gland.
8. Concerning pleomorphic adenoma (mixed salivary tumor), which of following
statements is incorrect:
a. is the commonest salivary tumor.
b. Has a very heterogeneous histological structure.
c. is well-encapsulated.
d. Presents as a slow-growing firm swelling just below the lobule of the ear.
e. Tends to recur after inadequate excision.
5) 6) 7) 8)
D E A C
24
9. Among the following statements about branchial fistula, the incorrect one is
that it
a. Commonly results from rupture of a branchial cyst.
b. Usually at the lower third of the anterior border of the sternomastoid muscle.
c. is often bilateral.
d. Discharges clear mucoid fluid.
e, Requires removal of the whole track by the "step-ladder' operation.
10. True statements about Ludwig’s angina do not include that it:
a. is a virulent cellulitis of the floor of the mouth.
b. Usually results from infection with staphylococci.
c. Causes marked swelling in the submandibular region with severe edema of the
tongue.
d. May cause suffocation.
e. May require urgent operation.
11. Langenbeek’s repair:
a- Is a method of grafting in burns
b- Is a method of repair after surgical excision of SCC
c- Is a method of repair of cleft lip
d- Is a method of repair of cleft palate
12. About cleft upper lip all are correct except:
o- Median is the most common
b- Lateral is due to non-fusion of median nasal process (future frenulum) with
maxillary process
c- Lateral is more common on left side
d- Best time for repair is the earliest possible time (3 months)
e- Repair is mainly for cosmetic purposes
9) 10) 11) 12)
A B D A
25
D E D E
26
17. The wrong statement about carcinoma of the lip is that it:
a. Affects males much more often than females.
b. Usually occurs on the lower lip.
c. May produce a "Kissing cancer on the other lip,
d. is most often a well-differentiated squamous cell carcinoma.
e. Spreads to the regional glands by lymphatic permeation.
18. A completely excised skin lesion of the face proved to be a basal cell
carcinoma. The further management of the case should be:
a. Lymph node dissection.
b. Radiotherapy.
c. Chemotherapy.
d. Regular follow up.
e. Reassurance of the patient.
19. Dental ulcer is characterized by the following features except that it:
a. Occurs in relation to a carious or irregular tooth.
b. Is usually painful.
c. Has an elongated shallow floor and a sloping edge.
d. Never causes enlargement of the regional lymph nodes.
e. May closely simulate malignant ulcer.
20. Carcinoma of the tongue infiltrating the mandible is best treated by:
a. Radiotherapy of both primary and regional gland.
b. Two-stage excision of primary and regional glands.
c. On block excision of primary and whole lymphatic area (Commando operation).
d. Radiotherapy for primary followed by radical neck dissection.
e. Excision of primary and radiotherapy to cervical lymph nodes.
B D D C
27
21. Concerning complete cleft palate, the untrue statement is that it:
a. is due to failure fusion of the palatal shelves of the maxillary processes with each
other and with the frontonasal process.
b. is often associated with cleft-lip and broadening of the face.
c. interferes with nutrition and speech.
d. Predisposes to upper respiratory tract infections.
e. Requires surgical repair after the second year of life.
22. The most frequent fracture of the face involves the:
a. Zygoma.
b. Moxilla.
c. Orbital floor.
d. Nasal bones.
e. Mandible.
23. Concerning fibrous epulis the incorrect statement is that it:
a. is a soft fibroma of the mucoperiosteum around a carious tooth.
b. Forms a small pedunculated swelling arising between two incisor teeth.
c. is covered by intact epithelium.
d. May be highly vascular or fibrous.
e. is treated by local excision.
24. Correct statements about dentigerous cyst include the following except that
it:
a. Occurs in children and adolscents in relation to a missing tooth.
b. is more common in the upper than in the lower jaw.
c. Presents as a globular swelling expanding the jaw.
d. Is lined with squamous epithelium.
21) 22) 23) 24)
E E E B
28
25. Which statement most accurately describes the anatomy of the sublingual glands?
a. They drain through the sublingual duct which opens into the floor of mouth.
b. They drain either directly on to the floor of mouth or into the submandibular duct.
c. They consist of two lobes separated by the mylohyoid muscle.
d. They are embedded in the intrinsic muscles of the ventral surface of the tongue.
e. They lie in the space between the mandible and the two bellies of digastric.
26. The term plunging ranula refers to which clinical entity?
a. A malignant congenital salivary mass arising from the submandibular gland
b. A benign salivary mass involving the parotid and submandibular glands
c. A mucous retention cyst originating from the sublingual glands, limited by the mylohyoid
muscle
d. A mucous retention cyst originating from the submandibular and sublingual glands which
perforates the mylohyoid muscle to enter the neck
e. A midline neck mass which moves on tongue protrusion.
27. Which of the following structures is not an anatomical relation to the submandibular
salivary gland?
a. The anterior facial vein
b. The facial artery
c. The inferior alveolar nerve
d. The lingual nerve
e. The hypoglossal nerve.
28. Which structure marks the posterior boundary of the submandibular duct which can
safely be accessed via an intra oral approach?
a. The third molar tooth
b.The body of the submandibular duct
c. The lingual nerve
d. The posterior edge of the mylohyoid
e. The marginal mandibular nerve
25) 26) 27) 28)
B D C C
29
29. Which structure attaches the deep lobe of the submandibular gland to the
lingual nerve?
a. The hypoglossal nerve
b.The submandibular ganglion
c. The deep cervical fascia
d. The tendon of digastric
e.The mylohyoid.
30. Which of the following is not a complication of submandibular gland
excision?
a. Frey’s syndrome
b. Anaesthesia of the ipsilateral tongue
c. Weakness of the corner of the mouth
d. Anaesthesia of submental skin
e. Paralysis of the ipsilateral tongue.
31. What percentage of submandibular tumors is malignant?
a. 20 per cent
b. 30 per cent
c. 40 per cent
d. 50 per cent
e. 60 per cent.
32. Which of the following is not a feature of salivary malignancy?
a. Facial nerve weakness
b. Rapid enlargement
c. Induration of the overlying skin
d. Cervical node enlargement
e. Rubbery consistency.
29) 30) 30) 31)
B A D E
30
33. Which is the most appropriate form of biopsy for a major salivary gland
tumour?
a. Open surgical biopsy to allow histology
b. Salivary washings
c. Frozen section during formal excision
d. Fine-needle aspiration cytology (FNAC)
e. Biopsy is contraindicated.
34. Which of the following structures does not lie in the parotid gland?
a. The facial nerve
b. Terminal branches of the external carotid
c. The glossopharyngeal nerve
d. The retromandibular vein
e. Lymph nodes.
35. What is the most common site for a parotid tumour?
a. At the anterior border of the masseter
b. Inferior to the angle of the mandible
c. As a parapharyngeal mass
d. Anterior to the ear
e. Behind the angle of the mandible.
36. Which nerve must be transacted as part of a superficial parotidectomy?
a. The facial nerve
b. The hypoglossal nerve
c. The greater auricular nerve
d. The accessory nerve
e. The auriculotemporal nerve.
37. Which of the following landmarks is used to locate the facial nerve trunk?
a. The insertion of sternomastoid
b. The greater horn of the hyoid
c. The superior-most portion of the cartilaginous ear canal
d. The insertion of digastric
e. The insertion of masseter.
45. About adenolymphoma, all the following statements are true, except:
a.The tumour is related to smoking
b.It may be bilateral
c.It has epithelial and lymphoid components
e.The tumour is cystic
e.Facial nerve palsy is characteristic of this tumour
46. All the following are possible complications of parotidectomy, except:
a.Loss of sensation on one side of the face
b.Inability to close the eyelids
c.salivary fistula
d.sweating on the side of the when eating(gustatory sweating)
e.Immobility of one angle of the mouth when patient smiles
47. About Ludwig’s angina, all the following statements are true, except:
a. This is tuberculous infection of the deep neck spaces
b.Diabetics are more prone to develop this condition
c.It causes laryngeal oedema
d. Oedema of the floor of mouth pushes the tongue upwards
e.Urgent incision and drainage is needed
48. A patient is diagnosed to have malignant metastasis in right upper deep
cervical lymph nodes. There are no symptoms to indicate the site of the primary.
The possible primary lesion may be any of the following, except
a. Nasopharyngeal carcinoma
b. Carcinoma of maxillary antrum
c. Papillary carcinoma of thyroid
d. Carcinoma of pyriform fossa of hypopharynx
e. Laryngeal carcinoma
45 46 47 48
E A A E
34
49 50 51 52
B E A A
35
53. About tracheostomy, all the following statements are true, except:
a. Tracheostomy is indicated if mechanical ventilation is needed for a long time
b. Emergency tracheostomy is done at the scene of an accident if there is upper
airway obstruction
c. Percutaneous tracheostomy is suitable for critically-ill ICU patients
d. In adults, an ellipse is removed from the anterior wall of 2nd, 3rd and 4th tracheal
rings
e.One of its possible complications is subglottic stenosis
54. Midline cystic swelling in the neck, moves up &down with deglutition and
with protrusion of the tongue:
a) Thyroid cancer.
b) Branchial cyst.
c) Ranula.
d) Thyroglossal cyst.
55. The commonest site for lip cancer is :
a) Angle of the mouth.
b) Lower lip at junction of middle, outer thirds.
c) Upper lip central part.
d) Upper lip at junction of middle, outer thirds.
56. The anterior 2/3 of the tongue receives common sensation through
…………nerve:
a) Glossopharyngeal.
b) Chorda tympani of facial.
c) Lingual.
d) Hypoglossal.
53 54 55 56
B B B C
36
a) Submandibular.
b) Parotid.
c) Sublingual.
d) Minor.
59. All of the following is correct about salivary glands’ stones except….:
a) Neck US may be a useful investigation.
b) 90% of them are radiopaque.
c) In bidigital examination, they are better felt from inside.
d) Pain may refer to tongue or teeth.
57 58 59 60
A A A C
37
61.A 24 years old female presented with a neck swelling on the left side, on
clinical examination it was located in the carotid triangle deep to the
sternomastoid muscle ,mobile from side to side ,it was transopaque on
transillumination, Aspiration revealed mucoid fluid rich in cholesterol crystals,
your provisional diagnosis is………..:
61 62 63 64
D c c b
38
65-A 62-year-old alcoholic presents with an indurated ulcer, 1.5 cm in length, in the left
lateral aspect of her tongue (not fixed to the alveolar ridge). There are no clinically abnormal
glands palpable in the neck, and a biopsy of the tongue lesion reveals squamous cell
carcinoma. What should she undergo?
(A) Chemotherapy
(B) Local excision of the ulcer
(C) Wide excision and left radical neck dissection
(D) Antibiotic therapy and should be encouraged to stop smoking
(E) Wide excision of ulcer and radiotherapy
66- A 59-year-old woman has discomfort in the posterior part of her tongue. Abiopsy
confirms that the lesion is a carcinoma. What is true in carcinoma of the posterior third of the
tongue?
(A) Lymphoid tissue is absent.
(B) Lymph gland spread is often encountered.
(C) There is an excellent prognosis.
(D) The tissue is well differentiated.
(E) The recurrent laryngeal nerve is infiltrated.
67-Adenocarcinoma is the predominant malignant lesion in which of the following?
(A) Hard palate
(B) Lip
(C) Anterior two-thirds of the tongue
(D) Larynx
(E) Esophagus
68-A 58-year-old woman undergoes excision biopsy of a tumor in the left posterior
triangle of her neck. Histology suggests that this is a metastatic cancer. What is the most
likely site of the primary tumor?
(A) Ovary
(B) Adrenal gland
(C) Kidney
(D) Piriform fossa
(E) Stomach
65 66 67 68
c b A D
39
69-The classic complete neck dissection for palpable adenopathy in the posterior
triangle of the neck includes removal of which of the following?
(A) The transverse process, C2–C4
(B) The spinal accessory nerve
(C) Both thyroid lobes
(D) The trapezius
(E) The vagus
70. A 40-year-old woman is suspected of having a carotid body tumor. Which
one of the following is most characteristic of such a tumor?
(A) They secrete catecholamines.
(B) They are more common at sea level.
(C) They arise from structures that respond to changes in blood volume.
(D) They arise from the structures that respond to changes in PO2.
(E) They are usually highly malignant.
71. Following surgical resection of a large thyroid mass, a patient complains of
persistent hoarseness and a weak voice. What is the most likely cause of these
symptoms?
(A) Traumatic intubation
(B) Prolonged intubation
(C) Injury to the recurrent laryngeal nerve
(D) Injury to the superior laryngeal nerve
(E) Scar tissue extending to the vocal cords
72. What is the most common site for foreign bodies in the head and neck?
(A) Eye
(B) Ear
(C) Nose
(D) Throat
(E) Esophagus
69 70 71 72
B D C B
40
73 74 75 76
B B A B
41
77 78 79 80
A D B D
42
81 82 83 84
C B A C
43
85 86 87 88
B B D D
44
89 90 91 92
B E C C
45
93 94 95 96
A D A B
46
97-Laryngocele is common to
a) Trumpet blowers
b) Glass blowers
c) Patient of chronic cough
d) All of the above
98-tuberculous cervical lymphadenitis commonly affects
a) Posterior triangle nodes
b) Upper deep cervical nodes
c) lower deep cervical nodes
d) submaxillary nodes
99-of the 800 lymph node of the body lymph nodes in the neck is aroud
a) 100
b) 200
c) 300
d) 400
100-cervical rib may present by which of the followings
a) Numbness in fingers
b) Bruit around clavicle
c) Lump in lower neck
d) All of the above
97 98 99 100
D B C D
47
Hernia
1. Which of the following is not a predisposing factor for a hernia?
a. Chronic obstructive pulmonary disease
b. Obesity
c. Urinary stones
d. Pregnancy
e. Peritoneal dialysis.
2. Which of the following are more common in multiparous women?
a. Indirect inguinal hernia
b. Lumbar hernia
c. Umbilical
d. Direct inguinal hernia
3. Which of the following is not a complication of an inguinal hernia?
a. Irreducibility
b. Inflammation
c. Strangulation
d. Obstruction
e. Bleeding.
4. Which of the following is not true in relation to strangulated
hernias?
a. They present with local and then generalized abdominal pain and
vomiting.
b. A normal hernia can strangulate at any time.
c. This is more common in femoral hernia.
d. They can be reliably excluded in irreducible hernias on clinical
examination.
e. They require urgent surgery.
1 2 3 4
c c e d
51
13- The following are True about the inguinal canal EXCEPT:
a. It is a 4 cm tunnel in the lower abdominal muscles.
b. It contains spermatic cord in Meals & Round ligament in female.
c. The floor is formed by the inguinal ligament & the roof is formed
by conjoint tendon.
d. Boundaries include external oblique muscle posterior & fascia
transversalis anteriorly.
14- Following organs can be found within the hernia sac EXCEPT:
a) Colon.
b) Ovary.
c) Urinary Bladder.
d) Prostate.
13 14 15 16
d d d b
54
18- All the following are True when the sac content in omentum
EXCEPT:
a) Doughy in consistency.
b) Gives No gurgling sensation.
c) Dull on percussion.
d) Reduced by difficulty in Beginning.
17 18 19 20
B D A c
55
24- About the internal ring test, all are true EXCEPT:
a) Can differentiate between indirect, direct & femoral hernia.
b) Not usually done.
c) Oblique hernia does not come out except after thumb release.
d) The test starts by asking the patient to reduce hernia.
21 22 23 24
b c d a
56
27- Regarding the direct hernia, all the following are true EXCEPT:
a) Neck of the sac is Medical to the inferior epigastric a.
b) The sac is within the cord including its contents.
c) Usually accuse in elderly and > 50 % is bilateral.
d) Un-commonly complicated.
25 26 27 28
C A B a
57
31- The most common type of hernia liable for strangulation is:
a) Incisional.
b) Para-umbilical.
c) Femoral.
d) Direct.
32- The commonest hernia type that can be seen strangulated is:
a) Incisional
b) Inguinal.
c) Para-umbilical.
d) Femoral.
29 30 31 32
D A C b
58
34- During the hernial repair, all the following are "wrong" except:
a. Viable intestine is returned back to the abdomen.
b. The omentum never to be excised.
c. The mesh should be applied to cover the cord.
d. Herniorrhaphy should be performed with good tension.
35- The ONLY cause of last expansile impulses on cough during hernia
examination is:
a) Irreducible hernia.
b) Obstructed hernia.
c) Strangulated hernia.
d) Retrosternal goiter.
33 34 35 36
B A C a
59
37 38 39 40
C C E D
60
41. Regarding the anatomy of the inguinal canal, which of the following
are true?
a. In infants the internal and external rings are almost superimposed.
b. The inferior epigastric vessels lie posterior and lateral to the internal
ring.
c. The inguinal canal is about 10 cm long and is directed downwards,
medially and forwards.
d. The internal ring is a U-shaped opening in the external oblique
aponeurosis 1.25 cm above the mid-inguinal point.
41 42 43 44
A B A A
61
45 46
E B
62
Testis
1 2 3 4
B C E d
63
5 6 7 8
B B E b
64
9 10 11 12
B D C d
65
16. The following statements about ectopic testis ore true except that
it:
a. is probably due to rupture of the scrotal tail of the gubernaculum
b. May lie in the groin or pubic region.
c. Does not develop normally.
d. Has normal long spermatic cord.
e. Can be easily replaced in the scrotum.
13 14 15 16
B A E c
66
17 18 19 20
D B A c
67
.
23-Precocious puberty is character of:
a. Sertoli cell tumor.
b. Leydig cell tumor.
c. Seminoma.
d. Teratoma.
21 22 23 24
B A B c
68
25 26 27 28
A C C b
69
29 30 31 32
C D B c
70
33 34 35 36
B E E e
71
37 38 39 40
E D D d
72
Anal Canal
1. Which of the following is false regarding anal canal anatomy?
a. The anorectal ring marks the junction between the rectum and the
anal canal.
b. The puborectalis muscle is concerned with the continence
mechanism.
c. The intersphincteric plane is not of much significance.
d. The internal sphincter is the thickened distal continuation of the
circular muscle layer of the rectum.
e. The dentate line is an important landmark representing the site of
fusion of the proctodaeum and postallantoic gut.
9 10 11 12
D A C b
75
13. The risk factors for anal squamous cell carcinoma (SCC)
include the following except:
a. Human papillomavirus (HPV) infection
b. HIV infection
c. Renal transplant
d. Other genital cancers
e. Rectal cancer.
13 14
E C
76
1 2 3 4
B C B E
77
5 6 7 8
A A A C
78
9.Which of the following statements(s) about gallstone ileus is/are not true?
o- The condition is seen most frequently in women older than 70.
b- Concomitant with the bowel obstruction, air is seen in the biliary tree.
c- The usual fistula underlying the problem is between the gallbladder and the
ileum.
d- When possible, relief of small bowel obstruction should be accompanied by
definitive repair of the fistula since there is of significant incidence of
Recurrence if the fistula is left in place.
e- Ultrasound studies may be of help in identifying gallstone of the obstructing
agent.
9 10 11 12
C E D C
79
13 14 15 16
E A C B
80
18. Which of the following lesions are believed to be associated with the
development of carcinoma of the gallbladder?
o- Cholecystoenteric fistula.
b- A calcified gallbladder.
c- Adenoma of the gallbladder.
d- Xanthogranulomatous cholecystitis.
e- All of the above.
17 18 19 20
A E E E
81
21 22 23 24
D B C C
82
25 26 27 28
A B C C
83
29 30 31 32
C D B C
84
33 34 35 36
B C D A
85
38. Rupture of on amoebic liver abscess occurs most often into the:
a. Peritoneal cavity.
b. Pleural cavity.
c. Pericardial cavity.
d. Duodenum.
e. Colon.
a. Emetine hydrochloride.
b. Metronidazole.
c. Aspiration.
d. Open drainage.
e. Excision.
.
37 38 39 40
C B D B
86
41 42 43 44
B A B B
87
46. Varceal bleeding not responding to drug and sclerotherapy is treated by:
a.Embolisation .
b.Surgical ligation.
c.TIPSS.
d.Liver transplant.
47. Which of the following procedures is associated with least risk of hepatic
encephalopathy?
a.Mesocaval shunt .
b. Proximol splenorenal shunt.
c. Distal splenorenol shunt' (Wonen shunt).
D. Side to side portocaval shunt.
48. A small cirrhotic liver with grossly enlarged caudate lobe demands
exclusion of:
a. Portal vein thrombosis.
b. Budd-Chiari syndrome.
c. Hepatoma .
d. Primary scleroising cholangitis
45 46 47 48
C C C B
88
49 50 51
A A A
89
Thyroid gland
3. As regard to T3:
a) Less concentration thon circulating T4.
b) More potent thon T4.
c) Mostly bound to TBG.
d) All of the above.
1 C
2 D
3 D
4 E
90
5 C
6 C
7 B
8 C
91
9 C
10 D
11 A
12 C
92
13 C
14 C
15 A
16 C
93
18. Which of the following treatment schedule for diffuse toxic goiter is
true:
a) Over 45yeors: radioactive iodine.
b) Under 45 years: with large goiter: surgery.
c) Under 45 years: with small goiter: antithyroid drugs.
d) None of the above.
e) All of the above.
17 C
18 E
19 A
20 B
94
21 D
22 D
23 B
24 C
95
25 E
26 D
27 E
28 E
96
29 D
30 B
31 B
32 E
97
33. A 17 year old girl presented with o 2.5 cm nodule in the right lobe
of the thyroid gland and enlarged three cervical LNs confirmed by US.
FNA cytology revealed malignant cells with vesicular nuclei, the most
probable diagnosis is:
a) Lymphoma
b) Anaplastic carcinoma
c) Follicular carcinoma
d) Papillary carcinoma
e) Medullary carcinoma
33 D
34 E
35 B
36 E
98
37 C
38 D
39 C
40 D
99
41 D
42 A
43 C
44 B
100
45 C
46 B
47 C
48 E
101
49 D
50 D
51 E
52 A
102
54- All the following are True about the thyroglossal cyst except:
a) Painless swelling.
b) Cystic in consistency.
c) Moves up & Down with swallowing but not with protrusion of
the tongue.
d) If infected, can form a thyroglossal fistula.
53 E
54 C
55 C
56 C
103
57 D
58 B
59 A
60 A
104
61- As differentiation between primary and 2ry toxic goiter all are true
EXCEPT:
a) The 2ry is more common in elderly.
b) The 2ry appears with gradual onset.
c) The 2ry is presented by diffuse enlargement of the gland.
d) Nervous manifestation, metabolic disorders and eye signs are
more prominent in the 1ry form.
64- Regarding the recurrent laryngeal nerve, all are true EXCEPT:
a) If injured bilaterally, completely, the patient will be aphonic.
b) Stridor will occur in unilateral complete injury.
c) Can be injured during devascularization of the inferior lobe.
d) Can be non-recurrent only on the right side.
61 C
62 C
63 D
64 B
105
67- The following is not one of the characters of the malignant thyroid
cyst:
a) Its aspirate is haemorrhagic.
b) Cytology shows malignant cells.
c) Rapid re-accumulation after aspiration.
d) The cyst will be completely disappeared after aspiration.
68- Malignant goiter, if painful the pain will be referred to the ear
through:
a) Phrenic nerve.
b) Sympathetic plexus.
c) Bronchial plexus.
d) Arnold's nerve ( branch of vagus nerve)
65 A
66 A
67 D
68 D
106
71- Hard goiter can be felt during examination of the gland in the
following cases:
a) Malignancy.
b) Calcified simple nodular goiter.
c) Riedle's thyroiditis.
d) Tense thyroid cyst.
72-All of the following are causes of dullness over the manubrium stain
EXCEPT:
a) Ectopic thyroid.
b) Pneumothorax.
c) Pre-tracheal lymph nodes.
d) Retrosternal goiter.
69 A
70 C
71 A
72 B
107
GIT
1 2 3 4
A B A C
108
5 6 7 8
B A B B
109
9- The least common site for cancer in the alimentary tract is:
a. Pharynx.
b. Esophagus.
c. Stomach.
d. Small bowel.
e. Colon and rectum.
11- The following are risk factors for volvulus of pelvic colon except:
a. Old age.
b. Chronic constipation.
c. Short sigmoid.
d. Adhesions.
9 10 11 12
D B C C
110
15- The most common cause of death in low intestinal obstruction is:
a. Toxemia.
b. Dehydration.
c. Electrolyte imbalance.
d. Generalized peritonitis.
13 14 15 16
A B A C
111
19- The following are recognized causes of paralytic ileus. The most
common is:
a. Diabetic ketoacidosis.
b. Drugs.
c. Peritoneal irritation.
d. Postoperative.
e. Spinal injury.
17 18 19 20
D D D D
112
23- Sigmoid volvulus has been associated with each of the following
except:
a. Chronic constipation and laxative abuse.
b. Chronic rectal prolapse.
c. Chronic traumatic paralysis.
d. Medical management of Parkinson's disease.
21 22 23 24
C C B C
113
26- A 20-year-old man swallowed two open safety pins. X-rays show
pins in the small intestine, the most appropriate management of this
point is:
a. IV antibiotics
b. 250 ml magnesium citrate orally
c. lmmediate surgery
d. Serial abdominal exam & x-rays if required
25 26 27 28
D D B E
114
29 30 31 32
A E D A
115
33 34 35 36
E D B C
116
37 38 39 40
B A A B
117
41 42 43 44
D C D A
118
45 46 47 48
D C E C
119
49 50 51 52
D E D C
120
55- Carcinoma of the left colon differs from that of the right colon in
the following except that it:
a. Is usually a stenosing scirrhous lesion.
b. Frequently presents with intestinal obstruction.
c. Rarely causes diarrhea.
d. Is usually impalpable on abdominal examination.
e. Carriers a better prognosis after radical resection.
56- The following statements about partial rectal prolopse are true
except that it:
a. Consists of a double layer of mucous membrane.
b. Is commonest in elderly people.
c. Is often associated with poor sphincter tone.
d. Is rarely associated with hemorrhoids.
e. Is best treated by ligature-excision of prolapsing mucosa.
53 54 55 56
C D C D
121
58- The best screening investigation for cancer of the colon in the
general population is:
a. Abdominal ultrasound.
b. Barium enema.
c. Stool occult blood test.
d. Carcinoembryonic antigen (CEA) assay.
e. Colonoscopy.
59. A 25-year-old male develops diarrhea and colicky abdominal pain.
Ulcertive colitis is diagnosed on colonoscopy. Which of the following
findings is consistent with the diagnosis?
(A) The rectum is not involved.
(B) The disease is confluent, there are no skip areas in the colon and the
rectum is involved.
(C) The full thickness of the bowel wall is involved.
(D) Microscopic examination of the mucosa reveals normal cells without
evidence of dysplasia.
(E) The incidence of colorectal cancer is equal to that of the general
population
60. A 35-year-old man has known ulcerative colitis. Which of the
following is an indication for total proctocolectomy?
(A) Occasional bouts of colic and diarrhea
(B) Sclerosing cholangitis
(C) Toxic megacolon
(D) Arthritides
(E) Iron deficiency anemia
57 58 59 60
A C B C
122
63. The standard initial therapy for acute sigmoid volvulus is:
(A) Laparotomy to reduce the volvulus and replace the sigmoid colon to
its normal position
(B) IV neostigmine
(C) Colonoscopy
(D) Ileostomy
(E) Rigid sigmoidoscopy
64. 3 cm in length. What is the most common complication of Meckel’s
diverticulum among adults?
(A) Bleeding
(B) Perforation
(C) Intestinal obstruction
(D) Ulceration
(E) Carcinoma
61 62 63 64
D A E C
123
65. A 30-year-old male is diagnosed with Peutz- Jeghers syndrome. What findings
are consistent with the diagnosis?
(A) Adenomas
(B) Hamartomas
(C) Adenomatous polyps
(D) Villoglandular polyps
(E) Villotubular polyps
66.A 79-year-old man has had abdominal pain for 4 days. An operation is
performed, and a gangrenous appendix is removed. The stump is inverted. Why
does acute appendicitis in elderly patients and in children have a worse prognosis?
(A) The appendix is retrocecal.
(B) The appendix is in the preileal position.
(C) The appendix is in the pelvic position.
(D) The omentum and peritoneal cavity appear to be less efficient in localizing the
disease in these age groups.
(E) The appendix is longer in these age groups
67. A 12-year-old boy complains of pain in the lower abdomen (mainly on the right
side). Symptoms commenced 12 hours before admission. He had noted anorexia
during this period. Examination revealed tenderness in the right iliac fossa, which
was maximal 1 cm below Mc Burney’s point. In appendicitis, where does the pain
frequently commence?
(A) In the right iliac fossa and remains there
(B) In the back and moves to the right iliac fossa
(C) In the rectal region and moves to the right iliac fossa
(D) In the umbilical region and then moves to the right iliac fossa
(E) In the right flank
68. On examination, patients presenting with appendicitis typically show maximal
tenderness over which of the following?
(A) Inguinal region
(B) Immediately above the umbilicus
(C) At a point between the outer one-third and inner two-thirds of a line between
the umbilicus and the anterior superior iliac spine
(D) At a point between the outer two-thirds and inner one-third of a line between
the umbilicus and the anterior superior iliac spine
(E) At the midpoint of a line between the umbilicus and the anterior superior iliac
Spine
65 66 67 68
B D D C
124
C A D C
125
E C A C
126
77 78 79 80
E A C A
127
81.A 55-year-old woman presents with pain in the LLQ of the abdomen
and fever of 102°F. On examination, she is found to be dehydrated and
has tenderness in the LLQ. A CT scan shows a mass in the LLQ involving
the sigmoid colon. There is a minimal amount of free fluid and no free
air. What should the initial treatment of this patient include?
(A) IV fluids, penicillin, and steroids
(B) IV fluids, cefoxitin, and nasogastric drainage
(C) IV fluids, blood transfusion, and laparotomy
(D) Immediate laparotomy
(E) Bowel preparation followed by Laparotomy
82. A 60-year-old man complains of recurrent attacks of painless rectal
bleeding. Colonoscopy reveals normal mucosa between the cecum and
the anal verge. What is the most helpful test to determine the cause of
bleeding?
(A) Angiography to look for angiodysplasia
(B) Technetium scan for Meckel’s diverticulum
(C) Upper GI endoscopy for peptic ulcer
(D) Small-bowel series for tumor
(E) Ultrasound for abdominal aortic Aneurysm
83. A 55-year-old man has had previous hemicolectomy for a
carcinoma of the right colon. At this time, 3 years after the primary
resection, a CT scan shows a solitary lesion in the right lobe of the liver.
What is the next step in management?
(A) Laser cauterization
(B) Radiotherapy
(C) Hepatic artery catheterization and local chemotherapy
(D) Symptomatic treatment with analgesics, because the colon disease is
now stage IV
(E) Exploratory laparotomy and resection of the tumor.
84. A 68-year-old man presents with crampy abdominal pain and
distention with vomiting. Findings on physical examination are positive
for healed abdominal scars. X-rays reveal multiple gas fluid levels. The
WBC count is 12,000. What is the most likely diagnosis?
(A) Small-bowel intestinal obstruction due to adhesions
(B) Hernia
(C) Appendicitis
(D) Inflammatory bowel disease
(E) Gallstones and ascites
81 82 83 84
B A E A
128
C A B A
129
89. Following resection of the left colon, a 67- year-old obese woman
develops left-sided leg edema due to deep-vein thrombosis. She is
placed on anticoagulants, but after 2 weeks of warfarin (Coumadin),
she develops a pulmonary embolus with slight hypoxemia. What
should the next step in management involve?
(A) Increasing the dose of anticoagulants
(B) Discontinuing anticoagulants
(C) Use of an inferior vena cava (IVC) filter
(D) CT scan of the leg and abdomen
(E) Femoral vein ligation
89 90
C B
130
91. A 45-year-old man complains of burning epigastric pain that wakes him up at
night. The pain is relieved by eating or using over-the counter antacids and H2
blockers. Diagnosis is best confirmed by which of the following?
(A) Urea breath test
(B) Serum gastrin levels
(C) Barium meal examination
(D) Upper endoscopy
(E) Upper endoscopy and biopsy
92. A 64-year-old woman with arthritis is a chronic NSAID user. She develops
severe epigastric pain and undergoes an upper endoscopy. She is told that she has
an ulcer adjacent to the pylorus. Which of the following is TRUE about the pylorus?
(A) It cannot be palpated at laparaotomy.
(B) It is not covered completely by omentum.
(C) It is a distinct anatomic entity that can
be distinguished during laparotomy.
(D) It is a true physiologic sphincter.
(E) It is a site where cancer is rarely found
93. A 30-year-old executive learns that he has a duodenal ulcer. His
gastroenterologist prescribes and outlines medical therapy. The patient worries
that if medical therapy fails he may need surgery. Which of the following is the
best indication for elective surgical therapy for duodenal ulcer disease?
(A) An episode of melena
(B) Repeated episodes of pain
(C) Pyloric outlet obstruction due to scar formation from an ulcer
(D) Frequent recurrences of ulcer disease
(E) Referral of pain to the back, suggestive of pancreatic penetration
94. A44-year-old dentist was admitted to the hospital with a 1-day history of
hematemesis caused by a recurrent duodenal ulcer. He has shown considerable
improvement following operative treatment by a truncal vagotomy and
pyloroplasty, 10 years prior to this incident. Which is TRUE of truncal vagotomy?
(A) It is performed exclusively via the thorax.
(B) It can be performed in the neck.
(C) If complete, it will result in increased acid secretion.
(D) It requires a gastric drainage procedure
(E) It has been abandoned as a method to treat ulcer disease
91 92 93 94
E C C D
131
95. A42-year-old executive has refractory chronic duodenal ulcer disease. His
physician has suggested several surgical options. The patient has chosen a parietal
(highly selective) vagotomy instead of a truncal vagotomy and antrectomy
because?
(A) It results in a lower incidence of ulcer recurrence.
(B) It benefits patients with antral ulcers the most.
(C) It reduces acid secretion to a greater extent.
(D) The complication rate is lower.
(E) It includes removal of the ulcer.
96. A63-year-old woman is admitted to the hospital with severe abdominal pain of
3-hour duration. Abdominal examination reveals board-like rigidity, guarding, and
rebound tenderness. Her blood pressure is 90/50 mm Hg, pluse 110 bpm (beats per
minute), and respiratory rate is 30 breaths per minute. After a thorough history
and physical, and initiation of fluid resuscitation, what diagnostic study should be
performed?
(A) Supine abdominal x-rays
(B) Upright chest x-ray
(C) Gastrograffin swallow
(D) Computerized axial tomography (CAT) scan of the abdomen
(E) Abdominal sonogram
98. Three months after recovery from an operation to treat peptic ulcer disease, a
patient complains that she has difficulty eating a large meal. A 99m Tc-labeled
chicken scintigraphy test confirms a marked delay in gastric emptying. A delay in
gastric emptying may be due to which of the following?
(A) Zollinger-Ellison syndrome (ZES)
(B) Steatorrhea
(C) Massive small-bowel resection
(D) Previous vagotomy
(E) Hiatal hernia
95 96 97 98
D B B D
132
101. A 63-year-old man has an upper gastrointestinal (UGI) study as part of his
workup for abdominal pain. The only abnormal finding was in the antrum, where
the mucosa prolapsed into the duodenum. There were no abnormal findings on
endoscopy. What should he do?
(A) Sleep with his head elevated.
(B) Be placed on an H2 antagonist.
(C) Undergo surgical resection of the antrum.
(D) Be observed and treated for pain accordingly.
(E) Have laser treatment of the antral mucosa.
102. A63-year-old man underwent gastric resection for severe peptic ulcer disease.
He had complete relief of his symptoms but developed “dumping syndrome.” This
patient is most likely to complain of which of the following?
(A) Gastric intussusception
(B) Repeated vomiting
(C) Severe diarrhea
(D) Severe vasomotor symptoms after eating
(E) Intestinal obstruction
103. A 65-year-old man was admitted to the hospital for severe bilious vomiting
following gastric surgery. This occurs in which circumstance?
(A) Following ingestion of gaseous fluids
(B) Spontaneously
(C) Following ingestion of fatty foods
(D) Following ingestion of bulky meals
(E) In the evening
106. A68-year-old woman has been diagnosed with a benign ulcer on the greater
curvature of her stomach, 5 cm proximal to the antrum. After 3 months of
standard medical therapy, she continues to have guaiac positive stool, anemia, and
abdominal pain with failure of the ulcer to heal. Biopsies of the gastric ulcer have
not identified a malignancy. The next step in management is which of the
following?
(A) Treatment of the anemia and repeat all studies in 6 weeks
(B) Endoscopy and bipolar electrocautery or laser photocoagulation of the gastric ulcer
(C) Admission of the patient for total parenteral nutrition (TPN), treatment of anemia, and
endoscopic therapy
(D) Surgical intervention, including partial gastric resection
(E) Surgical intervention, including total Gastrectomy
107. Over the past 6 months, a 60-year-old woman with long standing duodenal
ulcer disease has been complaining of anorexia, nausea, weight loss and repeated
vomiting. She recognizes undigested food in the vomitus. Examination and workup
reveal dehydration, hypokalemia, and hypochloremic alkalosis. What is the most
likely diagnosis?
(A) Carcinoma of the fundus
(B) Penetrating ulcer
(C) Pyloric obstruction due to cicatricial stenosis of the lumen of the duodenum
(D) ZES (Zollinger Ellison Syndrome)
(E) Anorexia nervosa
108. A 50-year-old woman presents with duodenal ulcer disease and high basal
acid secretory outputs. Secretin stimulated serum gastrin levels are in excess of
1000 pg/mL. She has a long history of ulcer disease that has not responded to
intense medical therapy. What is the most likely diagnosis?
(A) Hyperparathyroidism
(B) Pernicious anemia
(C) Renal failure
(D) ZES
(E) Multiple endocrine neoplasia
110. A 2-cm ulcer on the greater curvature of the stomach is diagnosed in a 70-
year-old woman by a barium study. Gastric analysis to maximal acid stimulation
shows achlorhydria. What is the next step in management?
(A) Antacids, H2 blockers, and repeat barium study in 6 to 8 weeks
(B) Proton pump inhibitor (PPI) (e.g., omeprazole) and repeat barium study in 6 to 8 weeks
(C) Prostoglandin E (misoprostol) and repeat barium study in 6 to 8 weeks
(D) Immediate elective surgery
(E) Upper endoscopy with multiple biopsies (at least 8 or 9) for the ulcer
111. A55-year-old school bus driver was diagnosed 3 months ago with an antral
ulcer. He was treated for H. pylori and continues to take a PPI. Repeat endoscopy
demonstrates that the ulcer has not healed. What is the next treatment option?
(A) Treatment with H2 blockers
(B) Vagotomy alone without additional surgery
(C) Endoscopy and laser treatment of the ulcer
(D) Distal gastrectomy with gastroduodenal anastomosis (Billroth I)
(E) Elevating the head of the bed when asleep
112. A 55-year-old man complains of anorexia, weight loss, and fatigue. AUGI study
demonstrates an ulcerated lesion at the incisura. Where is the incisura?
(A) Cardia
(B) Fundus
(C) Greater curvature
(D) Lesser curvature
(E) Gastrocolic ligament
113. A 36-year-old man presents with weight loss and a large palpable tumor in the
upper abdomen. Endoscopy reveals an intact gastric mucosa without signs of
carcinoma. Multiple biopsies show normal gastric mucosa. A UGI study shows a
mass in the stomach. At surgery, a 3-kg mass is removed. It is necessary to remove
the left side of the transverse colon. What is the most likely diagnosis?
(A) Gastric cancer
(B) Gastrointestinal stromal tumor (GIST)
(C) Choledochoduodenal fistula
(D) Eosinophilic gastroenteritis
(E) Linitis plastica
115. A63-year-old woman is admitted to the hospital with a UGI bleed that
subsides spontaneously within a short time after admission. A barium study shows
a gastric ulceration that is described by the radiologist as having a “doughnut sign.”
What is the most likely diagnosis?
(A) Lipoma
(B) Gastric ulcer
(C) Ectopic pancreas
(D) GIST
(E) Carcinoma
117. A 64-year-old woman presents with severe upper abdominal pain and
retching of 1-day duration. Attempts to pass a nasogastric tube are unsuccessful. X-
rays show an air-fluid level in the left side of the chest in the posterior
mediastinum. An incarcerated paraesophageal hernia and gastric volvulus is
diagnosed. What is the next step in management?
(A) Insertion of a weighted bougie to untwist the volvulus
(B) Elevation of the head of the bed
(C) Placing the patient in the Trendelenburg position with the head of the bed lowered
(D) Laparotomy and vagotomy
(E) Surgery, reduction of the gastric volvulus, and repair of the hernia
119. A 48-year-old man undergoes surgery for a chronic duodenal ulcer. The
procedure is a truncal vagotomy and which of the following?
(A) Gastroenterostomy
(B) Removal of the duodenum
(C) Closure of the esophageal hiatus
(D) Incidental appendectomy
(E) No further procedure
123. A 79-year-old retired opera singer presents with dysphagia, which has become
progressively worse during the last 5 years. He states that he is sometimes aware
of a lump on the left side of his neck and that he hears gurgling
sounds during swallowing. He sometimes regurgitates food during eating. What is
the likely diagnosis?
(A) Carcinoma of the esophagus
(B) Foreign body in the esophagus
(C) Plummer-Vinson (Kelly-Patterson) syndrome
(D) Zenker’s (pharyngoesophageal) diverticulum
(E) Scleroderma
124. A symptomatic patient has a barium swallow that reveals a 3-cm Zenker’s
diverticulum. The next step in management is?
(A) H2 blockers
(B) Anticholinergic drugs
(C) Elemental diet
(D) Bougienage
(E) Surgery (cricopharyngealmyotomy and diverticulectomy)
127. A 64-year-old man has symptoms of reflux esophagitis for 20 years. The
barium study shown (Fig. 5–2) demonstrates a sliding hiatal hernia. Which is TRUE
in sliding hiatal hernia?
(A) A hernia sac is absent.
(B) The cardiais displaced into the posterior mediatstinum.
(C) Reflux esophagitis always occur.
(D) A stricture does not develop.
(E) Surgery should always be avoided.
130. A 60-year-old man presents with excruciating chest pain. The pain follows an
episode of violent vomiting that occurred after a heavy meal. Subcutaneous
emphysema was noted in the neck. X-rays shows air in the mediastinum and neck,
and a fluid level in the left pleural cavity. What is the most likely diagnosis?
(A) Perforated duodenal ulcer
(B) Spontaneous rupture of the esophagus
(C) Spontaneous pneumothorax
(D) Inferior wall myocardial infarction
(E) Dissecting aortic aneurysm
131. A 45-year-old pilot has retrosternal burning; especially when he eats and lies
down to go to sleep. He has self-medicated himself with over the counter
heartburn medications. Upper endoscopy reveals an erythematous and inflamed
distal esophagus. In severe reflux esophagitis, the resting pressure of the LES is
decreased. This may be physiologically increased by which of the following?
(A) Pregnancy
(B) Glucagon
(C) Gastrin
(D) Secretin
(E) Glucagon
132. A46-year-old man has a long history of heartburn (GERD). His barium study
shows an irregular, ulcerated area in the lower third of his esophagus. There is
marked mucosal disruption and overhanging edges. What is the most likely
diagnosis?
(A) Sliding hiatal hernia with GERD
(B) Paraesophageal hernia
(C) Benign esophageal stricture
(D) Squamous carcinoma of the esophagus
(E) Adenocarcinoma arising in a Barrett’s Esophagus
133. A 46-year-old man present with dysphagia of recent onset. His esophogram
shows a lesion in the lower third of his esophagus. Biopsy of the lesion shows
adenocarcinoma. His general medical condition is excellent, and his metastatic
workup is negative. What should his management involve?
(A) Chemotherapy
(B) Radiation therapy
(C) Insertion of a stent to improve swallowing
(D) Surgical resection of the esophagus
(E) Combination of chemotherapy and radiation therapy
135. Following an emergency operation for hepatic and splenic trauma, the
surgeon inserts a finger into the foramen of Winslow in an attempt to stop the
bleeding. Which is TRUE of the hepatic artery?
(A) It is called the common hepatic artery at this level.
(B) It is medial to the common bile duct and anterior to the portal vein.
(C) It is posterior to the portal vein.
(D) It is posterior to the inferior vena cava.
(E) It forms the superior margin of the epiploic foramen.
137. A 44-year-old woman is scheduled for gastric surgery. She has no comorbid
disease. The anesthesiologist has difficulty inserting the orotracheal tube. In
between intubation attempts he uses an ambu-bag to oxygenate the patient. The
patient’s abdomen gets distended and tympany is noted in the left upper
quadrant. Suddenly the patient becomes hypotensive. Which of the following can
cause a vosogvagal response during anesthesia?
(A) The gastric remnant following a distal gastrectomy
(B) Corrosive gastritis
(C) Pernicious anemia
(D) Gastric volvulus
(E) Acute gastric dilatation
139. A50-year-old man is involved in a major motor vehicle collision and suffers
multiple traumas. He is admitted to the intensive care unit. After 2 days of hospital
admission he bleeds massively from the stomach. What is the probable cause?
(A) Gastric ulcer
(B) Duodenal ulcer
(C) Hiatal hernia
(D) Mallory-Weiss tear
(E) Erosive gastritis
141. A73-year-old woman is admitted to the hospital with a mild UGI hemorrhage
that stopped spontaneously. She did not require transfusion. She had ingested
large amounts of aspirin in the past 4 months to relieve the pain caused by severe
rheumatoid arthritis. Endoscopy confirms the presence of a duodenal ulcer. A
biopsy is done. What is the next step in the management of a duodenal ulcer
associated with a positive biopsy for H. pylori?
(A) H2 blockers
(B) Bipolar electrocautery of the ulcer
(C) Triple therapy
(D) Photocoagulation
(E) Elective surgery
142. A52-year-old artist develops epigstric pain that is relieved by antacids. She
also complains that her stool has changed color and is black and tarry. What is the
most important cause of the entity presenting above other than H. pylori?
(A) Submucosal islet cells
(B) Hyperglycemia
(C) Diet
(D) Acid secretion
(E) Acute erosive gastritis
143. An elderly patient delayed seeking medical attention for his early satiety and
weight loss because he attributed these changes to aging. When he underwent
upper endoscopy a large mass was seen in the stomach. Which statement is TRUE
regarding gastric carcinoma?
(A) During resection, it is safe to leave cancer at the cut edges.
(B) The incidence is increased in patients with gastric ulcer disease.
(C) Draining lymph nodes should not be removed.
(D) It is caused by diverticulitis.
(E) It is associated with hyperchlorhydria.
145. A patient has been diagnosed with achalasia. He refused surgery initially,
preferring to try nonoperative therapy. He tried life style modification, calcium
channel blockers, botulin toxin injection, and endoscopic pneumatic dilatation.
None of the treatments alleviated his symptoms. What are his surgical options?
(A) Esophagectomy
(B) Surgical esophagomyotomy proximal to the LES
(C) Modified Heller myotomy and partial fundoplication
(D) Repeat pneumatic dilation using pressures of loops
(E) Nissen fundoplication
146. A 50-year-old man presents to the emergency department with chest pain.
The patient is evaluated for a myocardiac infarction. The workup is negative. On
further questioning, his symptoms include dysphagia (with both liquids and solids).
Which of the following is TRUE?
(A) A barium swallow will always show a corkscrew esophagus.
(B) Manometry shows simultaneous high-amplitude contractions.
(C) Initial evaluation should exclude coronary artery disease.
(D) A pulsion diverticulum may be present.
(E) Patients refractory to medical management may respond to long esophagomyotomy.
147. A 60-year-old man has been having vague symptoms of upper abdominal
discomfort, early satiety, and fatigue. He is referred to a gastroenterologist, who
performs an upper endoscopy. Although a discrete mass is not visualized, the
stomach looks abnormal. It does not distend easily with insufflation. A biopsy
shows signet ring cells. Which of the following is TRUE?
(A) Signet ring cells are typically found in intestinal type gastric adenocarcinoma.
(B) Signet ring cell cancer is the most common type of gastric cancer.
(C) “Leather bottle stomach” is a term used to describe a nondistensible stomach infiltrated by
cancer.
(D) The gross appearance of the stomach always shows classic findings of linitus
plastica.
(E) Linitus plastica has an excellent prognosis.
150. A 54-year-old man presents with a massive UGI bleed. After resuscitation,
endoscopy is performed. No esophageal varices, gastritis, or gastric ulcers are
seen. After copious irrigation, a pinpoint lesion is seen near the GE junction.
What can be said about this lesion?
(A) It is a carcinoid.
(B) It is related to alcohol use.
(C) It is exclusively a mucosal lesion.
(D) Surgery if first-line therapy.
(E) Bleeding is from a submucosal vessel
152. A 70-year-old man has surgery for an abdominal aortic aneurysm. About 1
month later the patient presents with a massive UGI bleed. Which of the following
statements is TRUE?
(A) He should be given PPLs and observed in the intensive care unit.
(B) Most aortoenteric fistulas are primary.
(C) Most aortoenteric fistulas occur between the aorta and duodenum.
(D) It is not improtant to separate the aorta from the eosphagus after aortic surgery.
(E) This condition is always fatal.
151 152
B C
146
ORTHOPEDICS
a. Femur.
b. Tibia.
c. Clavicle.
d. Sternum.
a. Acidic.
b. Alkaline.
c. Neutral.
d. Any of the above.
a. Type of bone.
b. lntra-articular fracture.
c. Drugs.
d. Surgical intervention.
e. All of the above
1 2 3 4
C C B E
147
a. Angulations.
b. Rotation.
c. Elongation.
d. Shortening.
a. Unstable fractures.
b. Open fracture.
c. With neurovascular injuries (when exploration is needed).
d. Pathological fractures.
5 6 7 8
C C C B
148
a. Neurogenic shock.
b. Stroke.
c. Fat embolism.
d. Renal failure.
e. Sepsis.
a. Hypovolemic.
b. Neurogenic.
c. Carcinogenic.
d. Septic.
9 10 11 12
C E B C
149
a. Middle.
b. Medial end.
c. Between middle and lateral thirds.
d. Between middle and medial thirds.
13 14 15 16
C D D c
150
a. Posterior.
b. Anterior.
c. lnferior.
d. Superior.
a. Kocher's method.
b. Hippocrotic method.
c. Bonkort's operotion.
d. Arthrodesis.
17 18 19 20
C B C c
151
21 22 23 24
c d d b
152
25 26 27 28
c b d c
153
29 30 31 32
c c d d
154
33. Features seen on a child’s cervical spine XRay that are different
from an adult’s include all of the following EXCEPT
A Anterior pseudosubluxation C2 on C3 can be normal until 8 years
B Increased depth of anterior soft tissue space
C Predental space (C1) up to 5mm
D Prominent lordosis
35. Radial head fractures are commonly associated with all of the
following EXCEPT
A Lateral collateral ligament injury
B Medial collateral ligament injury
C Coronoid fracture
D Olecranon fracture
33 34 35 36
d d a d
155
37 38 39 40
c d c d
156
41 42 43 44
d b d c
157
45 46 47 48
e d c e
158
49 50 51 52
a a b a
159
53 54 55 56
c c d c
160
59. What is the key event required for a Triplane Ankle Fracture to
Occur?
A Inversion Injury
B Ossification of Epiphysis
C Axial Loading Injury
D Partial Fusion of Growth Plate
57 58 59 60
a b d d
161
61 62 63 64
c b a a
162
65 66
a a
163
Neurosurgery
1. A 43-year-old man experiences lower back pain after lifting a heavy object off the
ground. The following morning, he notices that the pain has begun to radiate down the
posterolateral aspect of the right leg and across the top of the foot to the big toe. The pain
is severe, electric in quality, associated with paresthesia over the same distribution, and
made worse by coughing. On examination, it is found that he has an area of diminished
sensation to pinprick over the dorsum of the right foot and mild weakness in his right
extensor hallucis longus muscle. The deep tendon reflexes are all intact. What is the most
likely diagnosis?
(A) Lumbar spinal fracture with compression of the caudaequina
(B) Herniated lumbar disk on the right at the level of L4–L5
(C) Herniated lumbar disk on the left at the level of L4–L5
(D) Herniated lumbar disk on the right at the level of S1–S2
(E) Intermittent claudication
2. A 48-year-old woman has a lower back pain and hypoesthesia in the left S1 dermatomal
distribution (left calf and lateral left foot). What is the most likely cause?
(A) A lesion at the right L4–L5 interspace
(B) Pathology where the nerve exits the spinal canal immediately above the pedicle of S3
vertebra
(C) A herniated nucleus pulposus
(D) Compression by the L5 lamina
(E) A lesion outside the vertebral column
3. A 35-year-old secretary complains of severe pain in the neck that radiates down the
right arm. The pain is electric in quality and affects specifically the radial aspect of the right
forearm and the thumb. She also describes numbness and paresthesia over the same
distribution. On physical examination, she is found to have an area of diminished
sensation to pinprick over the right wrist and thumb. The right biceps tendon reflex is
diminished, but there is no loss of muscle strength. She has right C5–C6 disk compression
and radiculopathy affecting which of the following?
(A) The right C4 root
(B) The right C4 mixed spinal nerve
(C) The right C4 anterior primary rami
(D) The right C6 root
(E) The right C6 spinal ganglion
6. A 63-year-old woman presents for workup to determine the reason for a gradual
hearing loss over approximately 5 years and intermittent tinnitus over the last
several months. Findings on physical and neurologic examination are
entirely within normal limits, except for the presence of sensorineural hearing loss
in the left ear. She has no cranial nerve deficits. An MRI of the brain with
gadolinium reveals the presence of an extra-axial tumor in the region
of the left cerebella-pontine angle. What is the most likely diagnosis?
(A) Epidermoid tumor (cholesteatoma)
(B) Glioblastomamultiforme
(C) Meningioma
(D) Acoustic neuroma
(E) Glomus tumor
5 6 7 8
c D E C
165
11. During her eighth month of pregnancy, a 29-year-old woman is noted to have
hydramnios. Further testing shows anencephalus. In this case hydramnios is
caused by which of the following?
(A) Impairment of the fetus’s swallowing mechanism
(B) Tumor of the fetus’s brain
(C) A secretory peptide from the placenta
(D) Excess antidiuretic hormone (ADH) from the fetus
(E) Renal agenesis
12. A 28-year-old man presents with a history of chronic headache. The headache
is intermittent, severe, poorly localized, and most often present when he arises in
the morning. He suffered a severe blow to the head and sustained a skull fracture
at the age of 15. Findings on his physical and neurologic examinations are within
normal limits. An MRI of the brain with gadolinium reveals the presence of a large,
nonenhancing extra-axial cyst in the region of
the right temporal tip. This most likely represents which of the following?
(A) An arachnoid cyst
(B) A cystic astrocytoma
(C) Rathke’s cleft cyst
(D) A Dandy-Walker cyst (failure of proper formation of the foramina of Lushka
and Magendie)
(E) Polycystic disease
9 10 11 12
B A A A
166
13. A 15-year-old boy complains of right-sided weakness and gait impairment. ACT
scan shows a large, nonenhancing cyst in the posterior cranial fossa, with an
enhancing tumor nodule in the left cerebellum. What is the most likely diagnosis?
(A) An arachnoid cyst
(B) A cystic astrocytoma
(C) Rathke’s cleft cyst
(D) Glioblastomamultiforme
(E) A large sebaceous cyst ans:B
14. A 43-year-old man is treated with pyridostigmine for facial, ocular, and
pharyngeal weakness due to myasthenia gravis. Which statement is true of
pyridostigmine?
(A) It is unrelated to neostigmine.
(B) It has far more side effects than neostigmine.
(C) Pyridostigmine and neostigmine reverse depolarizing neuromuscular blockade.
(D) It causes greater muscarinic effect than neostigmine.
(E) It is an anticholinesterase agentans:E
15. A 17-year-old male presents with 3-month history of headache, weight gain,
decreased concentration, polyuria, and polydypsia. His headaches are mostly in
morning and involvesthe frontal region. On examination he was found to have
bitemporal visual field defect and no facial hair. MRI scan revealed a suprasellar
partially calcified cystic lesion with displacement of optic chiasm. The most likely
pathology is:
(A) Giant aneurysm of carotid artery
(B) Pituitary macroadenoma
(C) Glioblastomamultiforme
(D) Craniopharyngioma
(E) Testicular metastasisans:d
13 14 15 16
B E D D
167
17. A 45-year-old woman was brought to emergency department for sudden onset
of severe headache associated with photophobia, nausea, and transient loss of
consciousness. On examination, she is awake and alert with normal cranial nerve
function. She also exhibits normal muscle strength and sensation.Her past medical
history is significant for sickle cell disease (SCD) and hypertension. CT scan confirms
the diagnosis of SAH without any intraparenchymal abnormality.
What is the least likely cause of SAH?
(A) Aneurysmal bleed
(B) Sickle cell angiopathy
(C) Arteriovenous malformation (AVM)
(D) Hemorrhagic meningioma
(E) Blood dyscrasia
17 18 19
D C E
173
Cardiothtoracic
l. About etiology of pneumothorax:
a- Spontaneous pneumothorax is always secondary to lung disease
b- Accidental pneumothorax needs a penetrating injury
c- Mechanical ventilation can result in pneumothorax
d- Central venous line insertion is not a reported cause
1. C
2. C
3. D
4. D
174
7. All of the following are differential diagnoses for sudden chest pain
without shock except:
a- Tension pneumothorax
b- Massive pulmonary embolism
c- Massive myocardial infarction
d- Flail chest
5. C
6 B
7 D
8 B
175
9 C
l0 A
11 A
12 E
176
14. All of the following ore indications of open surgical drainage of pus
in acute empyema except:
a- thick pus
b- Rapid recollection of pus
c- Streptococcal empyema
d- Extensive conditions
13 C
14 C
15 A
16 b
177
17 C
18 E
19 D
20 D
178
22. Open chest wounds are characterized by the following except that
they:
a. May cause "pendulum respiration".
b. May produce "mediastinal flutter.
c. May be associated with abdominal injuries.
d. Need immediate sealing of the wound.
e. Always require thoracotomy for definitive treatment.
21 E
22 E
23 A
24 E
179
25 A
26 D
27 A
180
Urology
1 2 3 4
b d b c
181
5 6 7 8
d c b a
182
9 10 11 12
b d c d
183
13 14 15 16
c b c e
184
17 18 19 20
b b d b
185
24. The triad of signs of rupture urethra includes the following except:
a. Perineal hematoma
b. Bleeding per urethra
c. Retention of urine
d. Bleeding per rectum
21 22 23 24
d c d d
186
25. Male 25 years old arrived hospital in severe shock after a road
traffic accident. Abdominal examination revealed tenderness & rigidity
in the loin and fullness in the suprapubic region. He failed to pass any
urine. He should be suspected lo have:
a. Rupture of the spleen
b. Rupture of the left kidney
c. Retroperitoneal hematoma
d. Extraperitoneal rupture of the bladder
e. lntrapelvic rupture of the urethra
25 26 27 28
b d e a
187
30. All of the following findings give sure diagnosis of renal TB except:
a. Acid fast, alchol fast bacilli on ZN stain.
b. Sterile pyuria.
c. +ve culture on lowenstein medium.
d. +ve culture on bactec medium.
29 30 31 32
e b c d
188
33 34 35 36
b b c b
189
37 38 39 40
e e b e
190
41 42 43 44
e b d a
191
45 46 47 48
c d c c
192
50. Renal calculus formation has been shown to be related to all of the
following except:
a. Vitamin D metabolism.
b. Urea splitting bacteria.
c. lmmobilization.
d. Stenosis of ureteropelvic junction.
e. Malabsorption.
49 50 51 52
d e e d
193
53 54 55 56
d a e b
194
57 58 59 60
e b d d
195
61 62 63 64
d d a c
196
65 66 67 68
d b b d
197
71. The formation of middle lobe in BPH arises from which zone:
a. Transitional zone
b. Central zone
c. Peripheral zone
d. None of the above
69 70 71 72
d c b d
198
75. The common malignant tumor of renal pelvis and ureter is:
a. Squamous carcinoma
b. Adenocarcinoma
c. Transitional cell carcinoma
d. Papillary carcinoma
73 74 75
d d c
199
Breast
1. One of the following statements as regard investigations for breast cancer is
true
A Fifty per cent of breast cancers are missed by population-based mammographic screening.
B Ultrasound can also be used as a screening tool.
C Magnetic resonance imaging (MRI) can be a useful imaging tool.
D Fine-needle aspiration cytology (FNAC) and core biopsy are equally useful diagnostically.
2. All of the following statements are true as regard benign breast disease,
except
A Benign breast disease is the most common cause of breast problems.
B Lipoma is a common condition of the breast.
C Traumatic fat necrosis can be mistaken for a carcinoma.
D 30 per cent of breast cysts recur after aspiration.
1 2 3 4
C B C d
200
5 6 7 8
B C C A
201
9. A 35-year-old premenopausal woman whose mother had breast cancer comes into
your office and has been told that she has fibrocystic breasts. On examination she
has multiple areas of thickening but no discrete mass. Of the following diagnostic
tests, which should be performed?
(A) Re-examination in 6 months
(B) Bilateral breast ultrasound
(C) Thermography
(D) Bilateral breast magnetic resonance imaging (MRI) with gadolinium
(E) Spot compression views if an area of discrete asymmetry or concerning calcifications is seen
11. After undergoing modified radical mastectomy for cancer of the right breast, a 52-
year-old female teacher becomes aware that the medial end of her scapula
becomes prominent in protraction movements at the shoulder. She also complains
of some weakness in complete abduction of the same shoulder. What nerve was
injured?
(A) Long thoracic
(B) Thoracodorsal
(C) Ulnar
(D) Intercostobrachial
9 10 11 12
D C A b
202
16.Concerning sampling of a breast lump, all the following statement are true,
except
a. The most accurate is excisional biopsy
b. Fine need aspiration cytology does not require general anaethesia
c. Fine need aspiration cytology shows tissue architecture.
d. Frozen section biopsy is sometimes inconclusive.
e. Trucut biopsy obtains a core of tissue.
13 14 15 16
B E E c
203
17.The following statements about lymphatic drainage of the breast are all true,
except
a. Lymph from medial half of the breast may drain in internal mammary
nodes.
b. From anywhere in the breast lymphatics drain mostly to axillary nodes
c. Division of axillary nodes into levels I, II and III depends upon the
anatomical relationship with the pectoralis minor.
d. Level one axillary nodes is the highest in the axilla.
e. The sentinel node is the earliest axillary node to be involved with cancer
spread.
20.For Modified radical mastectomy, all the following are true, except
a. The areola and nipple are removed
b. The whole breast tissue is removed
c. Block dissection of axillary nodes requires excision of axillary vein
d. Nerve to serratus anterior should be spared
e. The pectoralis major is spared.
17 18 19 20
D A D C
204
21.The following statements about post-mastectomy arm edema, all are true,
except
a. Is always transient
b. Adding radiotherapy to the axilla after clearance of lymph nodes increases
the risk of edema
c. Rough dissection of the axillary vein at surgery is one of the causes
d. Insertion of an IV line in the ipsilateral upper limb is a predisposing factor
e. May be caused by recurrence in the axilla.
22.When screening females for breast carcinoma, what if the most significant
risk
a. Three previous breast biopsies in premenopausal females.
b. More than 2 first degree relatives with ovarian or breast carcinoma
c. Hyperplasia in breast biopsy
d. None of the above.
24.The axillary lymph nodes are divided into three levels I,II, III by
a. The axillary artery
b. The axillary vein
c. The pectoralis minor
d. The clavicle
e. The nerve to latissimus dorsi
21 22 23 24
A B C C
205
27.How many lactiferous ducts drain into the nipple of the mature female
breast
a. 5-10
b. 15-20
c. 25-30
d. 35-40
25 26 27 28
A C B D
206
29 30 31 32
A C C B
207
33 34 35 36
C B A D
208
39.A 35 year old woman with BRAC-1 gene mutation seeks your advice about
her known increased risk of breast cancer. You should recommend
a. Mammograms and physical examination every 6 months until she is 50,
then bilateral prophylactic mastectomy
b. Mammograms and physical examination every 6 months + tamoxifen
c. Prophylactic bilateral mastectomy and if she has completed childbearing,
prophylactic bilateral oophorectomy.
d. None of the above.
37 38 39 40
B C C B
209
41 42 43 44
B D D D
210
45 46 47 48
D C A B
211
49 50 51
B A C
212
52. The most useful investigation for breast with prosthesis is:
a. Mammography.
b. US.
c. FNAB.
d. MRI.
e. Hormonal receptors.
52 53 54 55
d c d d
213
56 57 58 59
d d d c
214
60 61 62 63
c a b c
215
66. Risk factors of cancer breast include all the following except:
a- Mutation in suppressor genes (BRCA l, Il).
b- Early menarche.
c- Carcinoma in situ.
d- Prolonged lactation.
e- White races.
67. Increased occurrence of cancer breast in upper loteral quadrant is due to:
a- increased oestrogen receptors.
b- increased breast moss.
c- Both of the above.
d- None of the above.
64 65 66 67
b c d d
216
68 69 70 71
b c c e
217
b. may be asymptomatic
c. common on lt side
c. malignancy is suspected if the lump doesnot totally disappear after complete aspiration
72 73 74 75
C D B D
218
78. a women has 3 cm breast mass non palpable LNs and no clinical evidence of
distant metastasis, which is true?
a. stage II breast cancer
76 77 78 79
A E C b
219
c. mastectomy is indicated
80 81 82
C E A
220
GENERAL TOPICS
1. The most effective treatment of anaphylactic shock is :
a) lV anti histaminic.
b) lV corticosteroids.
c) lV antibiotics
d) lntracardiac adrenaline.
a) Penicillin injection.
b) Myocardial infarction.
c) Quinsy.
d) Loss of 8% blood volume.
e) None of the above.
a) CMV
b) Malaria
c) Hepatitis A
d) Brucellosis
1 2 3 4
B D C c
221
a) Trauma
b) Slipped ligature
c) infection
d) All of the above
6. The most urgent measure in the management of o severely
injured patient in the reception room is:
a) Massive antibiotics.
b) intravenous infusion.
c) Adrenocortical steroids.
d) Drainage of septic collections.
e) Vasopressors.
a) lntracranial hematoma.
b) Cerebrol-concussion.
c) internal hemorrhage.
d) Neurogenic shock.
e) Cardiogenic shock.
5 6 7 8
C d D c
222
a) Surgical wound.
b) intravenous sites.
c) Respiratory tract.
d) Urinary tract.
e) Deep veins of the leg.
9 10 11 12
D D C D
223
a) Pyrogenic reactions.
b) Thrombophlebitis of recipient vein.
c) Circulatory overloading.
d) incompatibility reactions.
e) Viral hepatitis.
a) Carpo-pedal spasms.
b) Positive Chovstek's sign.
c) Positive Trausseou's sign.
d) Tingling of fingers and circumoral region.
e) Defective blood coagulation,
a. Risus sardonicus.
b. Trismus.
c. Dysphagia.
d. Stridor.
13 14 15 16
D D D c
224
a. Wrist is flexed, MCP joints ore extended and lP joints ore flexed.
b. Wrist is flexed. MCP joints ore flexed and lP joints ore extended.
c. Wrist is extended, MCP joints ore extended and lP joints ore flexed.
d. Wrist is extended. MCP joints ore flexed and lP joints ore flexed.
e. Wrist is extended, MCP joints ore flexed and lP joints ore extended.
a. GlT.
b. Head ond neck.
c. Breast.
d. Lungs.
a. Acute gastritis.
b. Acute dilatation of the stomach.
c. Curling's ulcer.
d. Paralytic ileus.
e. Diarrhea.
17 18 19 20
B E B B
225
a. Zygoma.
b. Maxilla.
c. Orbital floor.
d. Nasal bones.
e. Mandible.
21 22 23 24
B E E c
226
25. E
227
PLASTIC SURGERY
1.Themain advantage of Lund and Browder charts over rule of nines is:
a- lt is easier
b- More accuroteto regarding sex of patient
c- More accuroteregarding age of patient
d- All of the above
1 2 3 4
C D C E
228
7.The most common site for the following diseases is correctly matched except:
a- Keloids :face, neck and skin over sternum
b- Hypertrophic scars :extensor surfaces
c- Hemangiomas:head and neck
d- Basal cell carcinoma:face
8. Langenbeek's repair:
a- lsa method of grafting in burns
b- lsa method of repair after surgical excision of Sq.C.C.
c- lsa method of repair of cleft lip
d- lsa method of repair of cleft palate
5 6 7 8
C E B D
229
9 10 11 12
A D C A
230
13 14 15 16
D A C B
231
20. A paraplegic bed-ridden patient developed a large deep bed-sore over his
sacrum. The best regimen is by wide excision and :
a. Local applications.
b. Thierschgrafting.
c. Wolfe-grafting.
d. Localskin flops.
e. Myocutaneousflap.
17 18 19 20
A B E E
232
21.lnunilateral hare-lip, the following statements are true except that it:
a. Affects the upper lip only.
b. Moy be partial or complete.
c. ls due to failure to fusion between the maxillary process with the fronto-nasal
process and the maxillary process of the opposite side.
d. lsalways associated with nasaldeformity.
e. Produces no serious speech defects.
24. A 6 year old boy has a 2 cm lesion on Right cheek. The lesion has been
enhancing since birth. On Auscultation lesion is pulsatile, it has a thrill and it has a
harsh sound on auscultation. It has multiple feeding arteries. The best treatment
option for this lesion would be
21 22 23 24
E E E D
233
a) Keloid is excessive scar formation within and outside the boundaries of skin incision
b) Recurence rate of keloid after excision is 10-20%
c) Exact cause of keloid formation is not known but fibroblast dysfunction is commonly
attributed
d) Adjunctive treatment besides surgery gives the best results
26. Which of the following is true about Langer's line of the skin?
25 26 27 28
B B B B
234
1 . The surface area of the peritoneal membrane is nearly equal to that of the skin.
T
2 . The parietal peritoneum is poorly innervated.
F
3. The peritoneum has the capacity to absorb large volumes of fluid.
T
4. The peritoneum has the ability to produce fibrinolytic activity.
T
5. When injured, the peritoneum produces an inflammatory exudate.
T
true?
30In a suspected foreign body (FB) in the oesophagus, water-soluble contrast examination should
be carried out.
F
31 When a food bolus is stuck in the oesophagus, always suspect an underlying disease.
T
32 Most iatrogenic perforations of the oesophagus can be treated conservatively.
T
33 Most spontaneous perforations of the oesophagus (Boerhaave’s syndrome) require an
236
operation.
T
34 In Mallory–Weiss syndrome the tear is usually in the lower end of the oesophagus
F
called
40 Whipple’s operation
F
41 Anderson–Hynes operation
F
42 Heller’s operation
F
43 Ivor–Lewis operation
T
44 Hartmann’s operation
F
47 The parietal cells are in the body of the stomach and are the acid-secreting cells.
T
48 The venous drainage of the stomach ends in the inferior vena cava (IVC).
F
49The chief cells secrete pepsinogen.
T
50 Gastric acid
F
51 H. pylori infection
T
52 NSAIDs
T
53 Smoking
T
54 High socioeconomic groups.
F
Which of the following are true with regard to the clinical features of
peptic ulcers?
55 The pain never radiates to the back and this differentiates this from biliary colic.
F
56 Vomiting is a notable feature.
F
57 Bleeding is rare.
F
58 They may cause gastric outlet obstruction.
T
59 Weight loss is a typical symptom.
F
cancer?
238
60 H. pylori infection
T
61 Gastric atrophy
T
62 Pernicious anaemia
T
63 Previous gastric surgery
T
64 Smoking.
T
of gastric cancer?
cancer?
gastric cancer?
80 Haematogenous metastases
T
81 Involvement of distant peritoneum
T
82 N3 nodal disease
F
83 Involvement of adjacent organs
F
84 Gastric outlet obstruction.
F
cancer?
240
100 The spleen lies in front of the left 10th, 11th and12th ribs.
F
101 The splenic artery arises from the coeliac axis.
T
102 The inferior mesenteric vein empties into the splenic vein.
T
103 The tail of the pancreas lies in the lienorenal ligament.
T
104 The inner surface of the spleen has two impressions – gastric and colic.
F
115 Patients with asymptomatic gallstones should routinely be advised to have a cholecystectomy.
F
116 Ninety per cent of patients with acute cholecystitis respond to conservative treatment.
T
117 Antibiotics are not required in the management of acute cholecystitis in the absence of
jaundice.
F
118 Urgent laparoscopic cholecystectomy in a patient with acute cholecystitis is associated with
a five times greater conversion rate compared with elective surgery.
T
119 Acalculous cholecystitis has a mild clinical course.
F
true?
120The incidence of symptomatic bile duct stones varies from 5 to 8 per cent.
T
121 Charcot’s triad consists of pain, stones and jaundice.
F
122 Primary sclerosing cholangitis (PSC) is associated with hypergammaglobulinaemia and
elevated smooth muscle antibodies.
T
123 Clonorchiasis can predispose to bile duct carcinoma.
T
124 Bismuth type 3 biliary stricture is a hilar stricture.
T
135 Ultrasonography (US) is the initial investigation of choice in the jaundiced patient.
T
136 When doing a computed tomography (CT) scan, initially an unenhanced scan must be done
followed by a scan after intravenous contrast injection (CECT).
T
137 While doing a magnetic resonance cholangiopancreatography (MRCP), intravenous secretin
injection helps to determine any obstruction to the pancreatic duct.
T
138 An increase in serum amylase is diagnostic of acute pancreatitis.
F
139 Endoscopic retrograde cholangiopancreatography (ERCP) should always be preceded by a
plain radiograph.
T
Pancreatic injury
244
145 All patients with pancreatic trauma should undergo an exploratory laparotomy.
F
146 Pancreatic duct disruption requires surgical exploration.
T
147 Severe injury to the duodenum and the head of the pancreas requires a
pancreatoduodenectomy.
T
148 After conservative management for pancreatic injury, duct stricture and pseudocyst may
occur as complications.
T
149 During splenectomy, iatrogenic injury to the pancreatic tail can occur.
T
150 Acute pancreatitis accounts for 3 per cent of hospital admissions in the UK for abdominal
pain.
T
151 Acute pancreatitis is classified into mild and severe.
T
152 80 per cent of cases are mild acute pancreatitis, with a mortality rate of 1 per cent.
T
153 20 per cent are severe acute pancreatitis, with a mortality of 20–50 per cent.
T
154 In all cases of acute pancreatitis, there is a marked rise in serum amylase.
F
165In the Western world, 60 per cent of the population over the age of 60 have diverticular
disease.
T
166A low-fibre diet causes the disease.
T
167 These diverticula consist of mucosa, muscle and serosa.
F5
168Those with perforation have a 10 times higher mortality than those with an inflammatory
mass.
T
169 Sepsis is the principal cause of morbidity.
T
185 In 95 per cent of cases, the disease starts in the rectum and spreadsproximally.
T
186 It is a diffuse disease affecting all the layers of the large bowel.
F
187 Granulomas are a typical microscopic feature.
F
188 The transverse colon is affected in toxic megacolon.
T
189 Patients may present as an emergency with fulminating colitis in 5–10 per cent.
T
complication of UC include
190 Carcinoma
T
191 Primary sclerosing cholangitis
T
192 Internal fistulae
F
193Ankylosing spondylitis
T
194 Perforation.
T
T
201 Severe dysplastic change or cancer on biopsy.
T
202 Non-compliance of medical treatment.
F
203Chronic steroid-dependent disease requiring large doses.
T
204 Extraintestinal disease.
T
205 In the emergency situation, total abdominal colectomy and ileostomy should be the
procedure of choice.
T
206 Proctocolectomy and ileostomy are associated with the lowest complication rate.
T
207 Restorative proctocolectomy with an ileoanal pouch should be considered in all patients.
T
208 Colectomy with ileorectal anastomosis is the most favoured procedure.
F
209 Ileostomy with a continent intraabdominal pouch is not often done.
T
About imaging in CD :
216 Barium enema and colonoscopy should be done for large-bowel disease.
T
217 MRI is the ‘gold standard’ for perianal fistulae.
T
218 CT scan is used for suspected intraabdominal abscess and internal fistulae.
T
224 20 per cent of FAP arise as a result of new mutations in the adenomatous polyposis coli
(APC) gene.
T
225 Large-bowel cancer occurs 15–20 years after the onset of the disease.
T
226 On surveillance, if there are no polyps by the age of 30 years, FAP is unlikely.
T
227 Polyps do not develop anywhere else other than the colon.
F
228 Colectomy with ileorectal anastomosis may result in rectal cancer later.
T
Colon cancer
T
330 Left colonic cancers present with rectal bleeding and obstructive
symptoms.
T
331 Even for an experienced colonoscopist, the failure rate to visualise the caecum is 10 per cent.
T
332 Intravenous urography (IVU) should be routinely done.
F
333 Synchronous cancers occur in 5 per cent.
T
Regarding intussusception:
417. In Glasgow coma scale, Eye opening scale is 1 to 4, Best verbal response scale is to 5. Best
motor response scale is 1 to 6. Thus, the lowest scale is 3, while the highest scale (fully
conscious is 15).
252
Ans. T
418. Muscles of the leg are enclosed in a tight sleeve that is formed of the deep fascia. Fascial septa
divide these muscles into two compartments.
Ans. F
419. Rigid fixation of a fracture is needed for healing of an arterial repair, otherwise it will be
disrupted or contused and thrombosed.
Ans. T
420. This patient is suspected to have injury of the bulbous urethra. If the patient voids, urine will
spread in the subcutaneous tissues, deep to the membranous part of the superficial fascia.
Ans. T
421. The most important factor that affects wound healing is vascularity of its edges.
Ans. T
422. A hernia must be repaired with absorbable material, so as to resist stresses for life.
Ans. F
423. Delayed primary closure of the skin means skin closure when pus stops to drain from the
wound.
Ans. F
424. Incised wounds treatment is by hemostasis, and leaving the wound open to drain.
Ans. F
425. The extent of tissue damage by a missile depended upon its weight.
Ans. T
Ans. T
427. Stab wound usually fracture bones and thus add to tissue damage.
253
Ans. F
428. Stab wounds are inflicted by knives and may injure viscera.
Ans. T
429. In the presence of contamination, delayed primary closure of an incised wound reduces the
possibility of infection.
Ans. T
430. In healing by secondary intention wound contraction and granulation tissue doas not play
major roles.
Ans. F
Ans. F
Ans. F
433. Metabolic acidosis is commonly seen in cases of shock though the pH is normal a low
bicarbonate level indicates metabolic acidosis
Ans. T
434. Adult respiratory distress syndrome (ARDS) is one example of the multiple organ failure
syndrome which occurs with shock.
Ans. T
435. Shock particularly the septic type major burns and severe pancreatitis can all trigger the
release of cytokines that lead to systemic inflammatory response syndrome(SIRS) the to
multiple organ failure (MOF).
Ans. T
Ans. F
Ans. F
Ans. F
439. A Fibroadenoma is a mixed epithelial and mesenchymal benign tumour. The peakage of
appearance is late teens and early twenties.
Ans. T
Ans. F
441. The commonest histological type of breast cancer is Ductal carcinoma in situ.
Ans: F
442. In breast cancer, T stands for tumour size.A tumour between 2-5 cm is T2.
Ans. T
443. Mammography indicated for regular screening of women who are at high-risk of
development of breast cancer.
Ans. F
444. Breast cancer is better for tumors in the medial half than the lateral one of the breast.
Ans. F
445. Paget’s disease of the nipple is lobular carcinoma arising at the opening of a lactiferous duct
on the nipple.
Ans. F
446. In a young female patient, a Lanz incision is preferred as this is more in keeping with
255
langer's lines.
Ans. T
448. A Collar incision is made for submandibular gland excision and block dissection.
Ans. F
449. Esophageal carcinoma is commoner in smokers, in people with increased alcohol intake
oesophageal strictures or certain dietary deficiencies.
Ans. T
450. A pharyngeal pouch occurs in a weakness in the superior constrictor muscle. It is also known
as killian's dehiscence.
Ans. F
Ans. T
452. In venous type ulcer the duplex show good arterial flow,so compression bandaging would be
suitable treatment.
Ans. T
453. In an arterial ulcer with stigmata of chronic arterial insufficiency, Revascularization should
be attempted, either endovascularly or surgically.
Ans. T
454. Inter-sphincteric abscesses can be incised, drained, and laid open. Trans-sphincteric
abscesses are usually laid open to reduce the risk of incontinenence following damage to both
internal and external sphincter mechanisms in this case seton drainage.
Ans. T
Ans. F
Ans. T
457. Epstein-Barr virus is associated with Burkitt's lymphoma in the presence of malarial
infection.
Ans. T
458. Aspergillus flavus releases afatoxin which occur as a dietary contaminants and are associated
with hepatocellular regeneration.
Ans. F
Ans. F.
Ans. T.
461. This man has a Glasgow coma scale (GCS) of 3.He is unable to protect his own airway;
securing the airway is therefore paramount
Ans. T
462. Hyperplasia is increased tissue or organ size due to increased cell size in response to
increased functional demand.
Ans. F
463. Atrophy is the decrease in size of an organ or tissue by reduction in cell size and /or reduction
in cell numbers.
Ans. T
464. Agenesis is the failure of development of an organ or structure. This differs from atresia
which is failure of development of a lumen in a normally tubular structure.
Ans. T
257
465. Transitional cell carcinomas of the renal pelvis resemble those affecting the bladder, but are
much less common they tend to invade the renal parenchyma and have a tendency to distal
spread.
Ans. T
466. HAEMORRHAGIC SHOCK class 1 is loss of 0-15% of circulating volume; there are no
obvious changes apart from the patient perhaps feeling uncomfortable and restless.
Ans. T
467. Painless jaundice together with a history of weight loss is strongly suggestive of a malignancy.
Ans. T
468. Sharing of needles among intravenous drug misusers is a serious risk factor for hepatitis B
and C both of which predispose to the development of hepatocellular cancer in later years
Alpha-fetoprotein is a marker for this cancer although it is also raised in hepatoma.
Ans. T
469. Colo-colic intussusception typically presents as in this case and the tumor acts as the apex.
Half of all adult intussusceptions are due to carcinoma.
Ans. T
Ans. F
Ans. T
472. A recent general anesthetic and opioid-based analgesia causing constipation are two factors
which predispose to developing urinary retention post-operatively.
Ans. T
Ans. T
474. Laparoscopic repair of inguinal hernias is recommended for the management of bilateral
inguinal hernias unless there are any contraindications present.
Ans. T
475. Squamous cell carcinoma is most common in sun-exposed areas frequently bleeds and is
characterized by an undermined edge.
Ans. T
476. Squamous cell characterized by a rolled pearly edge. It dose not tend to bleed as often as
melanoma.
Ans. F
477. Full-thickness skin grafting can be used on exposed tendon, but is not good for exposed bone
or cartilage.
Ans. T
478. Local flaps provide their own blood supply and can transfer skin, fascia, muscle, and bone.
Ans. T
479. Free flaps can not provides their own blood supply, and can be used with local flap.
Ans. F
480- Haemorrhoidal bleeding is painless and not mixed with stools it is bright red as its
origin from the arteriovenous cushions of the anal canal
Ans: T
481-Rubber band ligation is most useful in stopping bleeding from first and second degree
haemorrhoids for third degree haemorrhoids, band ligation is enough to control the prolapse
Ans: F
480.
482- Haemorrhoidectomy causes causes considerable postoperative pain. This may result in
reflex retention of urine,rather than incontinence
Ans: T
483-Faecal incontinence can occur if the whole anal sphincter mechanism is divided Anal stenosis
may follow haemorrhoidectomy if the surgeon leave intact muco-cutaneous bridges between the
259
raw areas
Ans: F
484-The majority of abscesses start as infection of an anal gland.These glands are present in the
intersphincteric space and have their ducts open at the dentate line
Ans: T
486- Surgery for a high fistula requires division of the entire anal sphincter mechanism and
would produce incontinence if done in one stage
Ans: T
487- The majority of anal canal cancers are of th squamous carcinoma type
Ans: T
488- The caudate lobe is segment I.it lies between the porta hepatic and the inferior vena
cava(IVC)posteriorly
Ans: T
489-The enzyme alkaline phosphatase is elevated in cases of bill stasis and with bone diseases
particularly bone resorption and destruction.
Ans: T
490-Being a retroperitoneal organ an isolated injury of the kidney is likely to stop bleeding
spontaneously because of the tamponade effect of the posterior peritoneum on the haematoma
Ans: T
491- The main danger of liver injury is bleeding However bleeding can stop spontaneously
Ans: F
492- Acute appendicitis always causes a liver abscess if it happents infection gets to the liver
through lymphatics
Ans: F
496- Viral hepatitis A resolves spontaneously without ill effects.One the hand types B and C can
cause liver cirrhosis and predispose to the development of hepatocellular carcinoma
Ans: T
497- Haemangiomas of the liver are common findings on imaging studies (CT or ultrasound)that
are done for other indications
Ans: T
498- carcinoembryonic abtigen is elevated in patients with GIT and breast cancer
Ans: T
500- The blood supply of HCC is the hepatic artery rather than the portal vein
Ans: T
501- Venous drainage of the testis is systemic haematogenous spread of testicular cancer goes to
the lungs rather than the liver
Ans: T